Ist die "Anzahl der Photonen" eines Systems eine Lorentz-Invariante?

Ich frage mich, ob die Anzahl der Photonen eines Systems eine Lorentz-Invariante ist. Google gibt ein Papier zurück , das darauf hinzuweisen scheint, dass es zumindest dann unveränderlich ist, wenn das System ein rechteckiger Hohlraum mit supraleitenden Wänden ist.

Mir wurde jedoch im hbar-Chatroom gesagt, dass es keine Invariante und proportional zum 1. Term des 4-Impulses ist, der mit dem Hamilton-Operator der "Freifeldtheorie" verwandt ist.

Heute habe ich mit einem Freund gesprochen, der etwas GR studiert (noch kein QFT) und er konnte nicht glauben, dass diese Zahl nicht Lorentz-invariant ist.

Alles in allem bin ich also verwirrt. Ist es für einige Systeme eine Lorentz-Invariante und für andere nicht? Wenn ja, welche Bedingungen muss ein System erfüllen, damit die Anzahl der Photonen invariant ist?

Eine gezählte Zahl ist eine gezählte Zahl, egal in welchem ​​Koordinatensystem Sie sie aufschreiben (Sie verwandeln sich nicht in Drillinge, indem Sie Ihren Zwilling in einer wirklich schnellen Rakete auf eine Weltraummission schicken) . Der erste Term in den vier Impulsen wäre eine Energie und transformiert sich natürlich unter einer Lorentz-Transformation. Das ist der Doppler-Effekt. Ich denke, das ist für den Fall der thermischen Photonen, die keine feste Anzahl haben, zunächst nicht ganz so trivial. Aussagekräftig ist nur die durchschnittliche Photonenzahl eines thermischen Zustands.
@CuriousOne Ich habe die Chat-Diskussion gefunden: chat.stackexchange.com/transcript/71?m=26894546#26894546 , es sieht so aus, als wären sich sowohl FenderLesPaul als auch ACuriousMind einig, dass es keine Invariante ist ... Ich würde gerne mehr darüber erfahren die Details.
Das würde ich nicht zu ernst nehmen. Es ist übrigens keine einfache Frage. Die Photonenzahl ist definitiv KEINE Invariante in beschleunigten Koordinatensystemen. Ich glaube nicht, dass es eine Invariante bei thermischen Photonen ist, die im thermodynamischen Gleichgewicht mit einem nicht-Lorentz-invarianten Thermalbad stehen müssen. Wenn Sie sieben Atome in ein festes Volumen werfen und sie sieben Photonen emittieren lassen, bleiben diese sieben nicht-thermischen Photonen jedoch in jedem Koordinatensystem immer sieben ... also ist es ein Ja, aber ...
Sieben detektierte Photonen wären für jeden Beobachter gleich; Es gibt keine Möglichkeit, etwas über Photonen zu wissen, ohne es zu entdecken.
Siehe core.ac.uk/download/files/425/11921990.pdf am Ende von Seite 9. Inäquivalente einheitliche Hilbert-Raum-Darstellungen einer QFT entsprechen unterschiedlichen Beobachtern, für die der Zahlenoperator unterschiedlich sein kann. Dies ist im Wesentlichen das Versagen des Stone-von-Neumann-Theorems. Siehe auch diese Antwort physical.stackexchange.com/questions/176652/…
@ user40276 Dieses Papier scheint den Unterschied zwischen Trägheits- und Nicht-Trägheitsrahmen anzusprechen. Aber soweit ich das beurteilen kann, widerspricht es nicht der Aussage, dass die Anzahl der Photonen in jedem Inertialsystem gleich ist und daher Lorentz-invariant ist.
@CuriousOne Das sollte mit leichter Erweiterung eine Antwort sein.
@JT: Ich lasse das von einem Theoretiker übernehmen. Für meinen Geschmack ist das zu viel Geschwätz bei einer wichtigen Frage. Ich bin sicher, man kann darauf eine viel bessere Antwort geben. Stimmt mein Intuitives mit dem richtigen Theoretischen überein, nehme ich einen Drink aufs Haus, will aber nicht behaupten, in einem Bereich, in dem ich es nur beflügeln kann, über ausreichende Expertise zu verfügen.
@Rococo Ja. Meine Behauptung war, dass allgemeine Beobachter eine andere Zahl feststellen werden. Natürlich können sie sich in völlig anderen Rahmen befinden, die nicht durch eine Lorentz-Transformation miteinander verbunden sind, oder sie befinden sich möglicherweise nicht einmal in einem Rahmen. Mein Punkt ist eher philosophisch als theoretisch. Der Fock-Raum ist nur eine der möglichen Darstellungen und daher führen unterschiedliche "Interpretationen" derselben QFT zu einem unterschiedlichen Zahlenoperator.
@Rococo Zum Beispiel, wenn wir zu einem Zeitpunkt zwei nicht äquivalente Darstellungen eines freien Systems zusammen mit Dynamik haben t in einem bestimmten Rahmen in einer Mannigfaltigkeit kann der Zahlenoperator in jedem System unterschiedliche Werte erreichen, aber sie sind nicht einmal vergleichbar, weil sie in verschiedenen Hilbert-Räumen leben.
@ user40276 okay, danke für die Klarstellung. Es ist sicherlich ein interessanter Punkt, ich muss mir das Papier, auf das Sie sich beziehen, genauer ansehen ...

Antworten (3)

Alice bereitet ein elektromagnetisches Feld in einem Zustand mit einer scharfen Anzahl von Photonen vor N ^ | n = n | n wo N ^ ist der Zahlenoperator. Alice wird in Bezug auf Bob verstärkt. In Bobs Bezugssystem befindet sich das Feld im Zustand U ^ ( Λ ) | n . Die Frage lautet, ob eine Messung der Anzahl von Photonen für Bobs Zustand die genaue Antwort gibt n . Mit anderen Worten, stimmt das N ^ U ^ ( Λ ) | n = n U ^ ( Λ ) | n ? Bob erhält das scharfe Ergebnis n wenn der Boost-Operator mit dem Zahlenoperator kommutiert. Wir müssen nur zeigen, dass der Kommutator [ U ^ ( Λ ) , N ^ ] = 0 .

Der Zahlenoperator für Helizitätsphotonen λ ist,

N λ ^ = d 3 p 2 ω η ^ p λ η ^ p λ
wo η ^ p λ , η ^ p λ sind Emissions- bzw. Absorptionsoperatoren für ein Impulsphoton p und Helizität λ (Die Notation für Emissions- und Absorptionsoperatoren stammt aus Diracs Monographie "Lectures on Quantum Field Theory"). Wir haben auch ω = p 0 im Lorentz-invarianten Maß.

Einzelne Photonenzustände transformieren sich als

U ^ ( Λ ) | p , λ = e ich θ ( p , Λ ) | Λ p , λ
wo θ ( p , Λ ) ist der Wigner-Winkel. Erzeugen eines Einzelteilchenzustands aus dem Vakuum | S durch | p , λ = η ^ p λ | S impliziert, dass die Emissionsoperatoren gleichartige Zustände transformieren,
U ^ ( Λ ) η ^ p λ = e ich θ ( p , Λ ) η ^ Λ p λ   .
Nehmen Sie das hermitische Konjugat, verwenden Sie die Einheitlichkeit und ersetzen Sie es Λ durch Λ 1 ,
η ^ p λ U ^ ( Λ ) = e ich θ ( p , Λ ) η ^ Λ p λ η ^ p λ U ^ ( Λ 1 ) = e ich θ ( p , Λ ) η ^ Λ p λ η ^ p λ U ^ ( Λ ) = e ich θ ( p , Λ 1 ) η ^ Λ 1 p λ   .
Bewerten Sie nun den Kommutator,
[ U ^ ( Λ ) , N ^ λ ] = d 3 p 2 ω U ^ ( Λ ) η ^ p λ η ^ p λ d 3 p 2 ω η ^ p λ η ^ p λ U ^ ( Λ ) = d 3 p 2 ω e ich θ ( p , Λ ) η ^ Λ p λ η ^ p λ d 3 p 2 ω η ^ p λ e ich θ ( p , Λ 1 ) η ^ Λ 1 p λ   .
Nehmen Sie eine Änderung der Variablen im zweiten Integral vor, p ' = Λ 1 p .
[ U ^ ( Λ ) , N ^ λ ] = d 3 p 2 ω e ich θ ( p , Λ ) η ^ Λ p λ η ^ p λ d 3 p ' 2 ω ' η ^ Λ p ' λ e ich θ ( Λ p ' , Λ 1 ) η ^ p ' λ
Der Wigner-Winkel θ ( p , Λ ) entspricht einer Rotationsmatrix R ( p , Λ ) = H Λ p 1 Λ H p wo H p ist der Standardschub. Jetzt,
R ( Λ p , Λ 1 ) = H Λ 1 Λ p 1 Λ 1 H Λ p = H p 1 Λ 1 H Λ p = ( H Λ p 1 Λ H p ) 1 = ( R ( p , Λ ) ) 1
damit der Wigner-Winkel θ ( Λ p , Λ 1 ) ist θ ( p , Λ ) . Beim Einsetzen dieses Ergebnisses in das letzte Integral verschwindet der Kommutator [ U ^ ( Λ ) , N ^ λ ] = 0 und so hat auch Bobs elektromagnetisches Feld die gleiche scharfe Zahl n von Photonen als Alices Feld.

Bearbeiten: Erklärung, warum das invariante Maß im Zahlenoperator erscheint

Die Methode der induzierten Darstellungen, die verwendet wird, um die Reaktion der einzelnen Teilchenzustände auf einen Lorentz-Boost (zweite Gleichung im Haupttext) zu erhalten, ist am einfachsten, wenn man ein Lorentz-invariantes Maß wählt, so dass die Auflösung für die einzelnen Teilchenzustände eins ist ist,

λ = ± 1 d 3 p 2 ω | p , λ p , λ | = 1   .
Diese Wahl impliziert, dass der Kommutator für die Emissions- und Absorptionsoperatoren ist
[ η ^ p λ , η ^ p ' λ ' ] = p , λ | p ' , λ ' = 2 ω δ λ , λ ' δ 3 ( p p ' )   .
Dies impliziert wiederum, dass der Hamilton-Operator normaler Ordnung für das freie elektromagnetische Feld ist
H ^ = 1 2 d 3 p ( η ^ p λ = 1 η ^ p λ = 1 + η ^ p λ = + 1 η ^ p λ = + 1 )   .
Jetzt erstellen n Photonen aus dem Vakuum mit einem Zustand,
| Ψ = ( η ^ p λ ) n | S
und verlangen, dass der Nummernbetreiber N ^ λ misst das scharfe Ergebnis n auf diesem Zustand. Dies impliziert, dass das Lorentz-invariante Maß in der Definition des Zahlenoperators (erste Gleichung im Haupttext) verwendet werden muss. Man sieht also, dass es hier keine Annahmen gibt, sondern nur eine Wahl des invarianten Maßes (anstelle eines quasi-invarianten Maßes), um die Methode der induzierten Darstellungen, die verwendet wird, um die Irreps der Poincare-Gruppe für masselose Teilchen zu erhalten, so einfach wie möglich zu machen .

Die große Annahme hier ist, wie sich die Fock-Raum-Zustände und Erzeugungsoperatoren in Bezug auf die Lorentz-Gruppe (dh
U ^ ( Λ ) | p , λ = e ich θ ( p , Λ ) | Λ p , λ
). Sobald diese Annahmen getroffen sind, ist das Ergebnis einfach. Sind diese Annahmen gültig? Da die Wellengleichung Lorentz-kovariant ist und ihre Lösungen Erzeugungsoperatoren bringen, würde ich sagen, dass diese Annahmen im freien Fall gültig und konsistent sind. Im interaktiven Fall ist es weniger klar. In einem beschleunigenden Frame zeigt der Unruh-Effekt deutlich, dass dies nicht funktioniert
Soweit über die nullte Komponente des 4-Impulses gesprochen wird, ist dies irrelevant. Niemand würde erwarten, dass der Hamiltonoperator Lorentz-kovariant ist. Natürlich würde es eine Rot/Blau-Verschiebung geben, aber das sagt nichts über die Anzahl der rot/blau verschobenen Teilchen aus, was Lorentz-invariant sein sollte, wenn wir die obigen Annahmen darüber treffen, wie sich der Fock-Raum transformiert.
@Saleh Hamdan: Die Reaktion der Einzelpartikelzustände auf einen Lorentz-Boost (zweite Gleichung in meiner Antwort) ist keine Annahme. Sie ist eine Folge des Auffindens der irreduziblen Darstellungen der Poincaré-Gruppe unter Verwendung der Methode der induzierten Darstellungen (siehe „Induced Representations of Groups and Quantum Mechanics“ von George W. Mackey, WA Benjamin, 1968) für den masselosen Fall. Diese Darstellungen werden nur für freie Partikel abgeleitet, und die Poincare-Gruppe arbeitet nur zwischen nicht beschleunigenden Referenzrahmen, wie Sie darauf hinweisen.
Die "Erklärung, warum das invariante Maß im Zahlenoperator erscheint" hat jeden Zweifel beseitigt. Vielen Dank
Wie stimmt die Antwort mit der akzeptierten Antwort von physical.stackexchange.com/questions/21830/… überein ?

Die Antwort eines Experimentators.

Es gibt unzählige Experimente, die zwei Gamma-Ereignisse messen. Die Lorentz-Invarianz ist eine Grundannahme für alle gemessenen Wechselwirkungen. Jede Wechselwirkung befindet sich in einem anderen Lorenz-Rahmen, abhängig von den beteiligten Energien und Impulsen. Wenn wir die Verteilungen von Querschnitten und Winkeln machen, sind wir von dieser Invarianz der Teilchenzahl in der beobachteten Wechselwirkung abhängig. Da es dem Standardmodell gelingt, all dies in sehr guter Näherung anzupassen, gilt diese Annahme.

Jetzt stammt jedes einzelne Photon aus einer Lorenz-invarianten Wechselwirkung durch Konstruktion elektromagnetischer Wechselwirkungen, obwohl nichts es aufzeichnet, also sollten die Zahlen konstant bleiben.

Wenn sich die Zahlen ändern, wenn sich das Lorenz-Koordinatensystem für ein Ensemble bereits erzeugter Photonen ändert, bedeutet dies, dass das auferlegte Lorenz-Koordinatensystem irgendwie mit den beobachteten Photonen interagiert. Wenn Energie ausgetauscht wird, können mehr Photonen erscheinen, was wie eine Nichterhaltung der Zahlen aussieht, aber nicht so betrachtet werden sollte.

Sie sprechen von Experimenten mit Gamma-Ereignissen, aber sie sind möglicherweise nicht zu empfindlich für zB einige Mikrowellen-Photonen, die in anderen Frames erscheinen.
@Ruslan Wenn die Produktion von Photonen, die Anzahl der Photonen, nicht Lorenz-invarian wäre, würden die berechneten Querschnitte, die aus der Hypothese der Lorenz-Invarianz stammen, nicht passen. Die Daten für den pi0-Zerfall zum Beispiel decken einen enormen Energiebereich des pi0 ab, jeder ist ein anderer Lorenzrahmen in Bezug auf die anderen Zerfälle. Mein Punkt ist, dass Sie zur Erzeugung eines Photons eine Wechselwirkung benötigen, sie werden nicht aus dem Vakuum erzeugt.

Ich bin mir nicht sicher, wer meine Kommentare, Argumente und Kritiker löscht, die Mr. Blakes Ableitung in Frage stellen. Nur weil wir nicht genug Argumente haben, um gegen eine Idee zu argumentieren, sollte dies niemals bedeuten, diese Stimme zu löschen. Soweit ich weiß, soll die Wissenschaft nicht monopolisiert werden, wie es auch beim Reichtum der Fall ist.

Beim Zählen der Anzahl von Photonen in einem System sind wir durch die Erkennung begrenzt, die wiederum durch das Heisenbergsche Unschärfeprinzip begrenzt ist, was in der Natur niemals überwunden werden kann. Theoretisch kann ein Photon über ein unendlich breites Spektrum jede Energie ungleich Null haben. Es besteht also eine Wahrscheinlichkeit ungleich Null, dass ein Photon eine bestimmte Energie hat. Vergessen wir nicht, dass Photonen nur ein Bestandteil der fundamentalen Teilchen in der Natur sind. Das bedeutet, dass sie mit mindestens jedem anderen geladenen Elementarteilchen des Standardmodells interagieren können, einschließlich ihrer Selbstwechselwirkungen. (Die Interaktionskanäle würden exponentiell zunehmen, wenn wir über Standardmodelltheorien wie Extradimensionen und Supersymmetrie hinausgehen). All dies sei gesagt, Es ist völlig in Ordnung, dass Photonen verloren gehen und/oder (durch ihre Wechselwirkungen) erzeugt werden und niemals in einem bestimmten endlichen Volumen unseres Universums (das als System bezeichnet wird) während einer endlichen Zeitspanne (innerhalb des endlichen Zeitalters des Universums) kompensiert werden. Da sie nur eine Teilmenge aller fundamentalen Teilchen sind und eine unendlich lange Lebensdauer haben, während der sie viele Wechselwirkungskanäle haben, wird ihre Anzahl in keinem endlichen Volumen unseres Universums erhalten bleiben. Es wäre jedoch schwieriger, die Frage zu verallgemeinern, „ob die Gesamtzahl aller fundamentalen Teilchen im sichtbaren Universum erhalten bleibt“. Um dies jedoch zu beantworten, Wir müssen die Theory of Everything kennen, in der ALLE fundamentalen Teilchen bekannt sind, einschließlich ihrer Lebensdauern und Massen und Interaktionskanäle, zusammen mit dem Wissen über Dunkle Materie und Dunkle Energie (die 96% des Universumsbudgets an Materie und Energie ausmachen, die ab sofort unsichtbar sind heute). Ich bin mir also sicher, dass die Antwort auf Ihre Frage lautet: "Nein. Die Anzahl der Photonen in einem endlichen Volumen, das als System bekannt ist, wird niemals erhalten bleiben." Und die anspruchsvollere möchte ich künftigen Generationen von Physikern überlassen.

Ich habe nicht abgelehnt. Aber Photonen sind nicht geladen, sagen Sie also nicht "jedes zweite geladene" Teilchen. Und konserviert ist anders als rahmeninvariant, sodass Ihre Antwort die Frage völlig missverstanden hat. Und das Universum muss sich nicht einmal in einem Zustand einer bestimmten Teilchenzahl befinden, und selbst wenn wir momentan wüssten , dass wir die Zahl der Teilchen ändern können, führen wir ständig Experimente und Beobachtungen durch, die dies beinhalten. Es ist also nicht konserviert, und die Frage war sowieso nach Frame-Invarianz, was völlig anders ist.
Ich bin mir nicht sicher, wer meine Kommentare, Argumente und Kritiker löscht, die Mr. Blakes Ableitung in Frage stellen. Nur weil wir nicht genug Argumente haben, um gegen eine Idee zu argumentieren, sollte dies niemals bedeuten, diese Stimme zu löschen. Soweit ich weiß, soll die Wissenschaft nicht monopolisiert werden, wie es auch beim Reichtum der Fall ist.
Ich habe Ihre Kommentare zu Stephen Blake nicht gelöscht, aber dies ist der zweite Kommentar von Ihnen, den ich gesehen habe, der als Antwort auf meinen Kommentar platziert zu sein scheint. Sind Sie sicher, dass Sie Blakes Antwort kommentieren? Und Kommentare sind ohnehin nicht für Diskussionen da, sie konzentrieren sich darauf, Antworten zu verbessern. Sie sind auch insofern zweitklassig, als es keine Erwartung gibt, dass sie überleben. Sie können hoch oder runter stimmen und das bleibt. Ein Kommentar soll zur Änderung einer Antwort anregen, und dafür ist er wirklich da. Aber Sie können keine Änderung einer Antwort erzwingen.
Hallo Timaeus, danke für den Hinweis. Ja, ich bin sicher, dass ich Blakes Antwort kommentiere. Ich weiß wirklich nicht, was hier los ist. Ich habe darauf gewartet, dass er mir antwortet, aber meine Frage scheint ständig gelöscht zu werden. Ich werde jedoch geduldiger sein, bis das Problem behoben ist. Aufrichtig,
Es steht Ihnen frei, selbst eine bessere Antwort zu finden.